Gortland has long been narrowly self-sufficient in both grain and meat. However,

游客2024-01-12  4

问题 Gortland has long been narrowly self-sufficient in both grain and meat. However, as per capita income in Gortland has risen toward the world average, per capita consumption of meat has also risen toward the world average, and it takes several pounds of grain to produce one pound of meat. Therefore, since per capita income continues to rise, whereas domestic grain production will not increase, Gortland will soon have to import either grain or meat or both.
Which of the following is an assumption on which the argument depends?

选项 A、The total acreage devoted to grain production in Gortland will not decrease substantially.
B、The population of Gortland has remained relatively constant during the country’s years of growing prosperity.
C、The per capita consumption of meat in Gortland is roughly the same across all income levels.
D、In Gortland, neither meat nor grain is subject to government price controls.
E、People in Gortland who increase their consumption of meat will not radically decrease their consumption of grain.

答案 E

解析 Argument Construction
Situation A country previously self-sufficient in grain and meat will soon have to import one or the other or both because its consumption of meat has risen as per capita income has risen. It takes several pounds of grain to produce one pound of meat.
Reasoning What conditions must be true for the conclusion to be true? Meat consumption is rising. What about grain consumption? A sharp reduction in the amount of grain directly consumed by meat eaters could compensate for increased meat consumption, making the conclusion false. If people did radically decrease their grain consumption, it might not be necessary to import grain or meat. Since the argument concludes that the imports are necessary, it assumes that direct consumption of grain by those who begin to eat meat will not plunge.
A The argument makes no assumptions about the acreage devoted to grain; it assumes only that the demand for grain will rise.
B The argument is based on rising per capita income, not population levels.
C The argument involves only meat consumption in general, not its distribution by income level.
D Since the argument does not refer to price controls, it cannot depend on an assumption about them.
E Correct. This statement properly identifies the assumption that those who begin to eat meat do not then greatly decrease their direct consumption of grains.
The correct answer is E.
转载请注明原文地址:https://tihaiku.com/zcyy/3357309.html
最新回复(0)